Last visit was: 24 Apr 2024, 12:05 It is currently 24 Apr 2024, 12:05

Close
GMAT Club Daily Prep
Thank you for using the timer - this advanced tool can estimate your performance and suggest more practice questions. We have subscribed you to Daily Prep Questions via email.

Customized
for You

we will pick new questions that match your level based on your Timer History

Track
Your Progress

every week, we’ll send you an estimated GMAT score based on your performance

Practice
Pays

we will pick new questions that match your level based on your Timer History
Not interested in getting valuable practice questions and articles delivered to your email? No problem, unsubscribe here.
Close
Request Expert Reply
Confirm Cancel
SORT BY:
Date
Tags:
Show Tags
Hide Tags
User avatar
Intern
Intern
Joined: 14 Oct 2006
Posts: 18
Own Kudos [?]: 158 [77]
Given Kudos: 0
Location: Boston
Send PM
Most Helpful Reply
User avatar
Retired Moderator
Joined: 16 Jun 2012
Posts: 871
Own Kudos [?]: 8554 [28]
Given Kudos: 123
Location: United States
Send PM
Tutor
Joined: 16 Oct 2010
Posts: 14817
Own Kudos [?]: 64898 [19]
Given Kudos: 426
Location: Pune, India
Send PM
General Discussion
User avatar
Manager
Manager
Joined: 04 Oct 2006
Posts: 72
Own Kudos [?]: 6 [0]
Given Kudos: 0
Location: Florida, Argentina, UK
Send PM
Re: In Gandania, where the government has a monopoly on tobacco sales, the [#permalink]
By spending less on health care, the gov. will cut an important source of debt in the LR. Therefore, A would weaken the argument.
User avatar
Manager
Manager
Joined: 14 Nov 2008
Posts: 108
Own Kudos [?]: 653 [0]
Given Kudos: 3
Concentration: Entrepreneurship
Schools:Stanford...Wait, I will come!!!
 Q51  V36
Send PM
Re: In Gandania, where the government has a monopoly on tobacco sales, the [#permalink]
winskc wrote:
In Gandania, where the government has a monopoly on tobacco sales, the incidence of smoking-related health problems has risen steadily for the last twenty years. The health secretary recently proposed a series of laws aimed at curtailing tobacco use in Gandania. Profits from tobacco sales, however, account for ten percent of Gandania’s annual revenues. Therefore, Gandania cannot afford to institute the proposed laws.

Which of the following, if true, most seriously weakens the argument?

A. All health care in Gandania is government-funded.
B. Implementing the proposed laws is not likely to cause a significant increase in the amount of tobacco Gandania exports.
C. The percentage of revenue Gandania receives from tobacco sales has remained steady in recent years.
D. Profits from tobacco sales far surpass any other single source of revenue for the Gandanian government.
E. No government official in Gandania has ever previously proposed laws aimed at curtailing tobacco use.


Only A & B are in contention. But it does not specify if what percentage of the tobacco..does the govt exports.. if it exports all, then, it also weakens.
But again..A is far more stronger choice for B to let in. :) I will go for A, what is OA.. ?
User avatar
Director
Director
Joined: 05 Jul 2008
Posts: 623
Own Kudos [?]: 1952 [2]
Given Kudos: 1
 Q49  V41
Send PM
Re: In Gandania, where the government has a monopoly on tobacco sales, the [#permalink]
2
Kudos
can't be D.

Conclusion is G cannot afford to institute the proposed laws that curtail the use of tobacco.

We need some thing that says G can afford to or have to afford.

Profits from tobacco sales far surpass any other single source of revenue for the Gandanian government, but still those profits make up only 10% of the revenues and the remaining 90% is untouched. Also it does not say how much will profits go down by to make such decision.

But with A, all HC funded by govt, govt can eliminate health costs and improve public health at the expense of profits from tobacco.

hence A
User avatar
Manager
Manager
Joined: 26 Oct 2008
Posts: 67
Own Kudos [?]: 232 [2]
Given Kudos: 0
Send PM
Re: In Gandania, where the government has a monopoly on tobacco sales, the [#permalink]
2
Kudos
Where do these questions with the strange OA's come from? icandy is correct: B (if it is relevant at all) would strengthen the argument, not weaken it. The correct answer is A.

As for D, the fact that profits from tobacco are the single greatest source of revenue tells us nothing at all about how much those profits would decline because of the laws, nor about how much could or could not be replaced by other revenue sources.

If anything, D might weaken the argument by showing that the WORST possible outcome would be to lose a maximum of 10% of revenues.
User avatar
Manager
Manager
Joined: 25 Oct 2012
Status:Final Lap
Posts: 190
Own Kudos [?]: 1284 [0]
Given Kudos: 85
Concentration: General Management, Entrepreneurship
GPA: 3.54
WE:Project Management (Retail Banking)
Send PM
Re: In Gandania, where the government has a monopoly on tobacco sales, the [#permalink]
nelz007 wrote:
in Gandania, where the government has a monopoly on tobacco sales, the incidence of smoking-related health problems has risen steadily for the last twenty years. The health secretary recently proposed a series of laws and aimed at curtailing tobacco use in Gandania. Profits from tobacco sales, however, account for ten percent of Gandania's annual revenues. Therefore, Gandania cannot afford to institute the proposed laws.

Which of the following,if true, most seriously weakens the argument?

A) All health care in gandania is government-funded. CORRECT because if u look more closely to this option, you can deducte from it that if the proposed laws were to be instituted, it will reduce smoking ---> health care GOOD ----> the goverment will compensate the profits from tobacco sales by the economy released through health care
B) Implementing the proposed laws is not likely to cause significant increase in the amount of tobacco gandania exports. Incorrect, we are not concerned with gandania exports
C) The percentage of revenue gandania recieves from tobaco sales has remained steady in recent years. Incorrect, no value added about whether gandania can or can not afford to institute the proposed laws
D) profits from tobaco sales far surpass any other single source of revenue for the gandandian government. Incorrect because if this is true, Gandania for sure, cannot afford to institute the proposed laws, hence this option strenghtens the conclusion
E) No government official in gandania has ever previously proposed laws aimed at curtailing tobacco use. Incorrect, so if this is true, how do we can figure out whether the proposed laws gonna work or not .

Could someone please explain the various answer choices. Thanks!


Hence, only A weakens properly the argument
User avatar
Intern
Intern
Joined: 20 Jun 2013
Posts: 4
Own Kudos [?]: 85 [0]
Given Kudos: 1
Send PM
Re: In Gandania, where the government has a monopoly on tobacco sales, the [#permalink]
Still don't understand why A is right?

Arguement: Profits from Tobacco = 10% of Gandania'a Revenue. So they should not implement this law.

A. All health care in Gandania is government-funded. <-- How does this come into picture? Isn't this Out of scope?

We can arrive at this by eliminating all other options but how is this justified?
avatar
Intern
Intern
Joined: 19 May 2014
Posts: 3
Own Kudos [?]: [0]
Given Kudos: 14
Send PM
Re: In Gandania, where the government has a monopoly on tobacco sales, the [#permalink]
For me also A - more smoker == more patients == more gov spending
Senior Manager
Senior Manager
Joined: 05 Feb 2018
Posts: 312
Own Kudos [?]: 794 [0]
Given Kudos: 325
Send PM
Re: In Gandania, where the government has a monopoly on tobacco sales, the [#permalink]
Premise:
1. healths problems from smoking up
2. proposed law = tobacco use down
3. tobacco profits = 10% of revenue
Conclusion:
weaken --> proposed law is not economically feasible


(A) All health care in Gandania is government-funded.
-- I actually marked it off but came back after the other ones were even worse... the justification is that they can afford to absorb the decrease in revenue from tobacco sales if they increase the costs of medical treatment.
(B) Implementing the proposed laws is not likely to cause a significant increase in the amount of tobacco Gandania exports.
-- if the modifier "not" wasn't present this would be a great answer. But, since their exports won't increase, they can't make up the monetary deficit from curtailing tobacco use. This strengthens the conclusion.
(C) The percentage of revenue Gandania receives from tobacco sales has remained steady in recent years.
-- irrelevant, doesn't help us determine if they can implement the law or not
(D) Profits from tobacco sales far surpass any other single source of revenue for the Gandanian government.
-- irrelevant, we aren't concerned with comparing the 10% profits
(E) No government official in Gandania has ever previously proposed laws aimed at curtailing tobacco use.
-- irrelevant, the history of legislation doesn't matter
Manager
Manager
Joined: 05 May 2019
Posts: 131
Own Kudos [?]: 556 [0]
Given Kudos: 143
Location: India
Send PM
Re: In Gandania, where the government has a monopoly on tobacco sales, the [#permalink]
The whole argument is a quant based argument. In the argument, the author is trying to say that the revenue of the country will decrease by curtailing the use of tobacco products. If we need to weaken the argument we need to find an option where there is some other source of revenue for the government.

(A) All health care in Gandania is government-funded. - After people have reduced their tobacco usage then they can become more health conscious and might seek for health institutes for their problems. If all health care in Gandania are government funded, they will generate revenue. We can keep this option and check other options
(B) Implementing the proposed laws is not likely to cause a significant increase in the amount of tobacco Gandania exports. - Even if the exports remain same, there is no way to increase the revenue for Gandania government.
(C) The percentage of revenue Gandania receives from tobacco sales has remained steady in recent years. - This option seems irrelevant to the argument or strengthen the fact that government will lose revenues
(D) Profits from tobacco sales far surpass any other single source of revenue for the Gandanian government. - By knowing the profits, we cannot know the effect in revenues unless we know the cost price as well. The government can increase the costs or might as well decrease them, we don't know.
(E) No government official in Gandania has ever previously proposed laws aimed at curtailing tobacco use. - We don't care about this

(A) should be the answer.
Manager
Manager
Joined: 09 Aug 2020
Posts: 226
Own Kudos [?]: 75 [0]
Given Kudos: 163
Location: India
Schools: IIMA PGPX'23
GMAT 1: 710 Q48 V39 (Online)
Send PM
Re: In Gandania, where the government has a monopoly on tobacco sales, the [#permalink]
This is such a beautiful question from GMAC !!
VP
VP
Joined: 11 Aug 2020
Posts: 1262
Own Kudos [?]: 201 [0]
Given Kudos: 332
Send PM
Re: In Gandania, where the government has a monopoly on tobacco sales, the [#permalink]
Grr. I duped myself out of the correct answer by coming up with two justifications for a

(A) All health care in Gandania is government-funded.

Why I didn't select this choice: One might argue that the conclusion is justified since Gandania would need the 10% of Gandania’s annual revenues that come from the profits from tobacco sales. In other words, they can’t afford to institute the laws given this constraint. This choice is out because it certainly doesn’t weaken, and perhaps might even strengthens the argument.

Why I should have selected this choice: This goes beyond the passage and is something that one should avoid, but the economic impact from disease burden can actually end up costing the government more than whatever they can gain from tobacco sales.

(B) Implementing the proposed laws is not likely to cause a significant increase in the amount of tobacco Gandania exports.

Well, if it doesn’t cause a significant increase in exports, then that means the proposed law would reduce the amount of revenue received. Isn’t that what we are trying to weaken?

(C) The percentage of revenue Gandania receives from tobacco sales has remained steady in recent years.

OK. That’s nice to know. This doesn’t however weaken the argument that Gandania cannot afford to institute the laws. It might even give us reason to think that if the laws are implemented that this trend of steady tobacco sales would break and hence the government would lose revenue – the very reason why the government can’t afford to institute the laws.

(D) Profits from tobacco sales far surpass any other single source of revenue for the Gandanian government.
This actually strengthens the argument. Now we know that not only are the profits from tobacco sales 10% of the total revenue but also that they are the largest source of such profits so if they lose out on this then it’s a problem for the government. One might be tempted to conclude here that since it’s the largest source of revenue that Gandania can in fact afford to institute the proposed laws. But, we don’t actually have any numerical information to support that here.

(E) No government official in Gandania has ever previously proposed laws aimed at curtailing tobacco use.

This is tangential to the argument. Would we be any worse off if we didn’t know this? No.

VeritasKarishma What do you think about the contradictory justifications in option A that I proposed? I was stuck on this for a half hour trying to find a sign of weakness.
Tutor
Joined: 16 Oct 2010
Posts: 14817
Own Kudos [?]: 64898 [1]
Given Kudos: 426
Location: Pune, India
Send PM
Re: In Gandania, where the government has a monopoly on tobacco sales, the [#permalink]
1
Kudos
Expert Reply
CEdward wrote:
Grr. I duped myself out of the correct answer by coming up with two justifications for a

(A) All health care in Gandania is government-funded.

Why I didn't select this choice: One might argue that the conclusion is justified since Gandania would need the 10% of Gandania’s annual revenues that come from the profits from tobacco sales. In other words, they can’t afford to institute the laws given this constraint. This choice is out because it certainly doesn’t weaken, and perhaps might even strengthens the argument.

Why I should have selected this choice: This goes beyond the passage and is something that one should avoid, but the economic impact from disease burden can actually end up costing the government more than whatever they can gain from tobacco sales.

(B) Implementing the proposed laws is not likely to cause a significant increase in the amount of tobacco Gandania exports.

Well, if it doesn’t cause a significant increase in exports, then that means the proposed law would reduce the amount of revenue received. Isn’t that what we are trying to weaken?

(C) The percentage of revenue Gandania receives from tobacco sales has remained steady in recent years.

OK. That’s nice to know. This doesn’t however weaken the argument that Gandania cannot afford to institute the laws. It might even give us reason to think that if the laws are implemented that this trend of steady tobacco sales would break and hence the government would lose revenue – the very reason why the government can’t afford to institute the laws.

(D) Profits from tobacco sales far surpass any other single source of revenue for the Gandanian government.
This actually strengthens the argument. Now we know that not only are the profits from tobacco sales 10% of the total revenue but also that they are the largest source of such profits so if they lose out on this then it’s a problem for the government. One might be tempted to conclude here that since it’s the largest source of revenue that Gandania can in fact afford to institute the proposed laws. But, we don’t actually have any numerical information to support that here.

(E) No government official in Gandania has ever previously proposed laws aimed at curtailing tobacco use.

This is tangential to the argument. Would we be any worse off if we didn’t know this? No.

VeritasKarishma What do you think about the contradictory justifications in option A that I proposed? I was stuck on this for a half hour trying to find a sign of weakness.



What you said would make sense had the argument said "the incidence of health problems has risen steadily for the last twenty years."
But what the argument actually says is "the incidence of smoking-related health problems has risen steadily for the last twenty years."
So we know that smoking is causing health problems.
Option (A) says that doctor's bills are paid by the Govt. So if smoking reduces, the incoming revenue may reduce but the incoming bills will reduce too. Hence cost to the govt will go down too. Then the govt may be able to afford passing the law.
User avatar
Non-Human User
Joined: 01 Oct 2013
Posts: 17213
Own Kudos [?]: 848 [0]
Given Kudos: 0
Send PM
Re: In Gandania, where the government has a monopoly on tobacco sales, the [#permalink]
Hello from the GMAT Club VerbalBot!

Thanks to another GMAT Club member, I have just discovered this valuable topic, yet it had no discussion for over a year. I am now bumping it up - doing my job. I think you may find it valuable (esp those replies with Kudos).

Want to see all other topics I dig out? Follow me (click follow button on profile). You will receive a summary of all topics I bump in your profile area as well as via email.
GMAT Club Bot
Re: In Gandania, where the government has a monopoly on tobacco sales, the [#permalink]
Moderators:
GMAT Club Verbal Expert
6919 posts
GMAT Club Verbal Expert
238 posts
CR Forum Moderator
832 posts

Powered by phpBB © phpBB Group | Emoji artwork provided by EmojiOne